star a is 5l ight years away an identiccal star of the absolute luminosity star b is 10 light years away compare tehir relative apparent brightness

Answers

Answer 1

In terms of the amount of light years energy received, star A should appear 100 times brighter than star B. This is due to the fact that received energy diminishes according to squared distance.

How far apart are two stars from one another?

Contact binaries are binary star systems with members whose distance from one another is so great that their outer envelopes actually touch. This structure can actually be rather stable, with typical lifetimes measured in millions or even billions of years, which may surprise you.

How do we figure out how far away the stars are from Earth?

When the Earth is at the other end of the star's orbit, six months later, astronomers take another measurement of the star's position. This results in a base line that is equal to the sum of the two angles used to measure the star and the double distance from Earth to the Sun's center (about 300,000,000 km). The distance to a nearby star can be calculated using these three variables in a very straightforward trigonometric manner.

Learn more about Stars here:

brainly.com/question/7379319

#SPJ4


Related Questions

A water wave is called a deep-water wave if the water’s depth is more than one-quarter of the wavelength. Unlike the waves
we’ve considered in this chapter, the speed of a deep-water wave depends on its wavelength:
v = Bgl2p Longer wavelengths travel faster. Let’s apply this to standing waves. Consider a diving pool that is 5.0 m deep and 10.0 m wide. Standing water waves can set up across the width of the pool. Because water sloshes up and down at the sides of the pool, the boundary conditions require antinodes at x = 0 and x = L. Thus a standing water wave resembles a standing sound wave in an open-open tube.
a. What are the wavelengths of the first three standing-wave modes for water in the pool? Do they satisfy the condition for being deep-water waves?
b. What are the wave speeds for each of these waves?
c. Derive a general expression for the frequencies fm of the possible standing waves. Your expression should be in terms of m, g, and L.
d. What are the oscillation periods of the first three standing wave

Answers

The oscillation periods of the first three standing wave is 3.58 sec.

Deep-sea waves lie deeper than half a wavelength. A water wave is a shallow water wave if the depth is less than the wavelength divided by 20.

calculation:-

a. The wavelength of the first three standing waves are V = √g h m/2π

b. V1 = 5.59, V2 = 3.95, V3 = 3.22 m/s

C. A general expression for the frequencies fm of the possible standing waves. Your expression should be in terms of m, g, and L frequency = 1/lamda m × √g h m/2π.

d. The oscillation periods of the first three standing wave are T = 1/f

= √4πL/ng

putting the values t = 3.58

t2 = 2.52 sec and t3 = 1.25 sec.

Deep-sea waves do not interact with the seafloor on their way so their velocity is independent of water depth. However, when the wave enters shallow water, the interaction with the bottom changes the wave. The wave speed slows down the wavelength shortens and the wave height increases.

They bend in paths that are more perpendicular to the water surface propagate more slowly and decrease in wavelength as they enter shallow water. So when a wave of water travels from deep water to shallow water it slows down shortens its wavelength and changes direction.

Learn more about The wave speeds here:- https://brainly.com/question/12215474

#SPJ4

find the volume of a gas in liters if 2.95 mol has a pressure of 0.760 atm at a temperature of 52oc. group of answer choices

Answers

The volume of a gas in liters if 2.95 mol has a pressure of 0.760 atm at a temperature of 52oc is 116.584.

volume of a gas = 2.95 × 0.760 × 52 = 116.584

We understand temperature as the average translational kinetic energy linked to the disordered motion of atoms and molecules, and the kinetic temperature as derived from kinetic theory provides reasonable accuracy. In light of the fact that higher energy molecules are colliding with lower energy molecules and transferring energy to them, it follows naturally that the flow of heat is from a region of high temperature toward a region of lower temperature. Since temperature only measures the kinetic energy portion of internal energy, it is not directly proportional to internal energy, so two objects with the same temperature do not necessarily have the same internal energy (see water-metal example). The three commonly used temperature scales are used to measure temperatures (Celsius, Kelvin, and Fahrenheit).

Learn more about temperature here:

https://brainly.com/question/1191204

#SPJ4

the speed of a bobsled is increasing because it has an acceleration of 2.4 m/s2. at a given instant in time, the forces resisting the motion, including kinetic friction and air resistance, total 450 n. the combined mass of the bobsled and its riders is 270 kg. (a) What is the magnitude of the force propelling the boblsed forward?
(b) What is the magnitude of the net force that acts on the bobsled?

Answers

 The speed of a bobsled is increasing because it has an acceleration of 2.4 m/s2. at a given instant in time 1363.6N.

Describe what acceleration is.

Acceleration refers to the rate at which the velocity and direction of a moving object change over time. Anything is said to be sped when it goes straighter and faster or slower. Therefore, even though the speed is constant, travel on a circle increases due to the constant change in direction.

In what ways do velocity and velocity differ?

The rate at which a displacement varies is measured by its velocity. Speed changes occur at an accelerated rate. Its magnitude and orientation are both present, making it a vector quantity. As the rate at which velocity changes, acceleration is likewise a vector quantity.

To know more about acceleration visit:

https://brainly.com/question/11936480

#SPJ4

5. What is the mass of a car that its going 2 m/s² and then hits a tree with a force of 6000N
GIVEN
WORK
ANSWER:3000kg

Answers

Answer:

Below is the explanation

Explanation:

You use Newton's second law which states:

Force (N) = mass (kg) x acceleration (m/s squared)

So we substitute:

6000 = ? x 2

Rearrange so:

? = 6000/2

which = 3000 kg

a ball is thron upward witha an intial velocity of 61 feet per second from an initial height of 12 feet express the heigs in feet of the ball as a function of the time t (insecond ) how long will the ball be in the air

Answers

The time at which the ball remains in the air after it is thrown vertically upward with an initial velocity of 61 feet per second from an initial height of 12 feet is 4 seconds.

Motion that is normal to a defined horizontal surface is often referred to as vertical motion. The height of a ball launched from a certain height while maintaining a constant speed is modeled as h(t) = at²+bt+c where h(t) is the height of the ball, t is time, and a, b, and c are constants.

At the initial height h = 12 feet and t = 0,

12 = a(0)²+b(0)+c

c = 12

Differentiating equation h(t) = at²+bt+c concerning t, we get, [tex]\frac{dh(t)}{dt}&=2at+b[/tex]

Substituting t = 0 and dh(t)/dt = 61, we get,

[tex]\begin{aligned}61&=2a(0)+b\\b&=61\end{aligned}[/tex]

Again differentiating [tex]\frac{dh(t)}{dt}&=2at+b[/tex] concerning t, we get, [tex]\frac{d^2h(t)}{dt^2}=2a[/tex]

The acceleration caused by gravity on Earth is 32 feet per second. Since the ball travels downward, the value becomes negative. Substituting this in the above equation.

[tex]\begin{aligned}-32&=2a\\a&=-16\end{aligned}[/tex]

Substituting values of a, b, and c in h(t) = at²+bt+c, we get,

h(t) = -16t²+61t+12

Substituting h = 0 in the above equation, the time the ball remains in the air will be,

0 = -16t²+61t+12

0 = 16t²-61t-12

Using the quadratic formula,

[tex]\begin{aligned}t&=\frac{-(-61)\pm\sqrt{(-61)^2-4\times16(-12)}}{2\times16}\\t&=\frac{61\pm67}{32}\\t&=\frac{61+67}{32}\;\text{or}\;\frac{61-67}{32}\\t&=4\;\text{or}\;-\frac{3}{16}\end{aligned}[/tex]

Time cannot be negative, therefore, t = 4 seconds is the answer.

To know more about the vertical motion of the ball:

https://brainly.com/question/29115368

#SPJ4

you place a small air cart on a 3.00 m air track and set it into motion with a speed of 0.70 m/s. due to very good elastic bumpers at each end of the track, the cart oscillates back and forth between the two ends of the track with essentially no loss in energy. determine the period and the frequency for this motion. you may assume that the length of the cart is negligible compared to the length of the track.

Answers

The motion has an 8.57 second period, and its frequency is 0.11 Hz. You can suppose that the distance between the cart and the track is little.

Period= T = 2*3/0.7 = 8.57 sec

The period of the motion is 8.57seconds

 freq = f = 1/T = 1 /8.57 = 0.11 Hz

The frequency of the motion is0.11Hz

The number of waves that pass a fixed point in a unit of time is known as frequency in physics. It is also the number of cycles or vibrations that a body in periodic motion experiences in a unit of time. When a body in periodic motion moves through a series of events or locations before returning to its initial state, it is said to have experienced one cycle or one vibration. A simple harmonic motion is also seen under angular velocity.

If one cycle or vibration takes half a second to complete, the frequency is two per second; if it takes a full hour, the frequency is one hundred per hour. Frequency is often equal to the reciprocal of the period, or time interval; for example, frequency = 1/period = 1. (time interval).

Learn more about frequency here:

https://brainly.com/question/14316711

#SPJ4

A 25 cm rod moves at 6.8 m/s in a plane perpendicular to a magnetic field of strength 0.12 T. The rod, velocity vector, and magnetic field vector are mutually perpendicular, as indicated in the accompanying figure.(a) Calculate the magnetic force on an electron in the rod (In N). (Enter the magnitude.) N (b) Calculate the electric field in the rod (in V/m). (Enter the magnitude.) V/m (c) Calculate the potential difference between the ends of the rod (in V). (d) What is the speed of the rod (in m/s) if the potential difference is 1.0 V? m/s

Answers

Option B is correct. The cross product of velocity and magnetic field, q [v B], determines the strength of the force.

The magnetic field's direction is anticipated by the right-hand thumb rule, and as a result, the resultant force is perpendicular to both the velocity and magnetic field directions.

Using the known currents in the wires, the force per unit length can then be computed as follows: Fl=(4107Tm/A)(5103A)2(2)(5102m)=11010N/m.

Given, \sV=200V\sv=100m/s

e=1.6×10 \s−19 \s C

m=9.1×10 −31 \s Kg \sm \se \s​ \s = \s9.1×10 \s−31

1.6×10 \s−19

m \se \s​ \s =1.75×10 \s11

The best choice is B.

Learn more about magnetic here-

https://brainly.com/question/3160109

#SPJ4

A fisherman has caught a fish on his pole, and to keep the pole steady he has to apply an upward force of F2 = 240 N at an angle of 83.5° with respect to the pole (see figure). The length of his pole is 4.1 m, and he is holding it a distance 0.35 m from the end, where he is applying a downward force F1.
With how much force, F1, in newtons, does he have to push straight downward on the end of his pole to keep the pole from moving? You may assume the pole is massless. F1 = What is the mass of the fish on the end of the pole, in kilograms? m =

Answers

The mass of the fish on the end of the pole, in kilograms m =2.090Kg.

What is Force?

Force is an influence that can cause an object to change its motion, direction, shape, or other physical attributes. Forces can be applied to objects, creating a change in their acceleration, velocity, or momentum. Forces can be either contact forces, such as pushing or pulling on an object, or non-contact forces, such as gravity, magnetism, and electricity. Forces can be described in terms of magnitude, direction, and point of application. Magnitude is the strength of the force, direction is the path the force is applied, and point of application is the point on the object where the force is applied. Forces can also be classified as balanced or unbalanced. Balanced forces occur when two forces of equal magnitude and opposite direction act on an object, resulting in no net force. Unbalanced forces occur when two forces of unequal magnitude act on an object, resulting in a net force.

Given that,

F2 = 240N

Angle = 83.5 degree

Length = 4.1m

Distance = 0.35m

How much force, F1 in newtons ?

What kind of fish is that hanging off the end of the pole?

Thus,

Calculating Torque about ‘A’

For equillibrium ∑τa= 0

F1 * L + F2 * (L – d) = 0

F1 * L= - F2* (L – d)

F1L SinΘ= F2(L – d) SinΘ

F1 = F2 (L – d)/L

F1 = 240 (4.1 – 0.35)/4.1

F1 = 219.51N

For equillibrium ∑ Fnet = 0

F2 + F1 + mg = 0

mg = F2- F1

m = F2 - F1/ g

m = 2.090Kg

To learn more about Force:

https://brainly.com/question/12970081

#SPJ1

a student spins a cup that is in contact with a platform in a horizontal circular path at a constant speed, as shown in figure 1. the platform is connected to two strings that are held by the student as the platform-cup system experiences uniform circular motion. a free-body diagram of the cup is shown in figure 2. which statement of reasoning best supports and correctly identifies the magnitude of the force the cup exerts on the platform?

Answers

√F1²+F2², because this force reflects the overall force the cup has applied to the platform.

What factors contribute to circular motion?

The change in direction and centripetal force. A centripetal force is applied to any item travelling in a circle (or along a spherical route). Towards other words, the item is being physically pushed or pulled in the direction of the circle's center. This is the necessary centripetal force.

What exactly is the force pulling on a particle traveling in a circle?

Centripetal force is the name of the force exerted on a particle that is rotating in a circle. It is this centripetal force that is focused inward that prevents the particle from deviating from or losing its circular course.

To know more about Centripetal forces visit

brainly.com/question/11324711

#SPJ4

What does an oscilloscope measuring electric current display
A. How resistance changes over time
B. How power changes over time
C. How current changes over time
D. How voltage changes over time

Answers

D.how voltage changes over time
Ddddddddddddddddddddddddddddddd

A 12 kg runaway grocery cart runs into a spring with spring constant 220 N/m and compresses it by 80 cm .
What was the speed of the cart just before it hit the spring?

Answers

The speed of the cart just before it hit the spring is 3.42 m/s.

Elastic potential energy is the energy that is stored when a force is applied to deform an elastic object.

Given the mass of the cart is 12 kg, the spring constant is 220 N/m, and the compressed length is 80 cm.

Given that the runaway grocery cart compresses the string. When this cart stops, the kinetic energy (KE) of this cart becomes zero because of maximum compression. This KE will be converted to the elastic potential energy of the spring. Then,

[tex]\begin{aligned}U&=\frac{1}{2}kx^2\\&=\frac{1}{2}\times220\times0.80^2\\&=\mathrm{70.4\;J}\end{aligned}[/tex]

This elastic potential energy is equal to the initial KE.

[tex]\begin{aligned}70.4&=\frac{1}{2}\times 12\times v^2\\140.8&=12v^2\\v^2&=11.73\\v&=\mathrm{3.42\;m/s}\end{aligned}[/tex]

The answer is 3.42 m/s

To know more about elastic potential energy:

https://brainly.com/question/29311518

#SPJ4

while cruising at 190 knots, you wish to establish a climb at 160 knots. when entering the climb (full panel), it would be proper to make the initial pitch change by increasing back elevator pressure until the

Answers

You want to start a climb at 160 knots while cruising at 190 mph. It would be reasonable to perform the initial pitch change upon starting the climb (full panel) by increasing the rear elevator pressure until the Attitude indicator displays.

Once the knots are set, you can adjust the pressure until the pitch attitude indicator approximates the pitch attitude required for the 160-knot climb. The approximate pitch attitude needed for the 160-knot climb. You want to start a climb at 130 knots while cruising at 160 knots. It is proper to make the initial pitch change while starting a climb (full panel) by increasing the back elevator pressure until the attitude indicator displays the approximate pitch attitude needed for a 130-knot ascent. You want to start a climb at 130 knots while cruising at 160 knots. It is proper to adjust the initial pitch while commencing the climb (full panel) by raising the back elevator pressure till the end.

To learn more about  elevator pressure please click on below link

https://brainly.com/question/29665127

#SPJ4

A student calculates the time taken (in minutes) for the light from the sun to reach the Earth. The sun is 93 million miles away (1 mile 1.609 km) and the speed of light is 3.00 x 108 m/s. The calculation is set up as follows: 9.3 x 10^7 mi x 1609 km/1 mi x 10^3m/1 km x 1 s/3.00 x 10^8 m x 1 min/60s Select the correct answer, quoted to the correct number of significant figures. O 8.3 min, since 9.3 x 10 has 2 significant figures O 8.313, since 1.609 has 4 significant figures O 8 min, since there is only 1 significant figure in 60 s O 8.31 min, since 3.00 x 10° has 3 significant figurts

Answers

The time taken (in minutes) for the light from the sun to reach the Earth is 8.3 minutes.

The speed of sunlight is 300,000 kilometres per second. The basic equation "time= (distance/speed)" can be used to determine how long it takes for sunlight to reach Earth.

We are given that,

Speed of light = c = 3×10⁸ m/s

Distance from sun to earth = d = 93 million miles = 148.8× 10⁹m

Thus to calculate the time of sun light which is reached to earth by the equation,

t = d / c

t = (148.8 × 10⁹m)/( 3×10⁸ m/s)

t = 496 sec

t = 8 min 3 sec.

Therefore , the sunlight reached to earth in 8min 3sec.

To know more about time

https://brainly.com/question/8012273

#SPJ4

Two round concentric metal wires lie on a tabletop, one inside the other. The inner wire has a diameter of 22.0 and carries a clockwise current of 15.0 , as viewed from above, and the outer wire has a diameter of 40.0. What must be the direction (as viewed from above) of the current in the outer wire so that the net magnetic field due to this combination of wires is zero at the common center of the wires? What must be the magnitude of the current in the outer wire so that the net magnetic field due to this combination of wires is zero at the common center of the wires?

Answers

The current in the outer wire is in the counterclockwise direction.

The concept of the right-hand thumb rule and magnetic field at the center of a current carrying loop is required to solve the problem.

First, determine the direction of the current in the outer wire by using the right-hand thumb rule. Then, calculate the magnitude of current in the outer wire by equating the sum of magnetic fields due to each wire to zero.

The magnetic field at the center of a current loop is, B = [tex]\mu[/tex]₀I/2R

Here, I is the current through the loop, R is the radius of the loop, and

[tex]\mu[/tex]₀ is the permeability of free space.

The right-hand thumb rule is used to find the direction of the magnetic field. Curl the fingers of the right hand into a half circle in such a way that the thumb points in the direction of the current, then the fingers will point in the direction of the magnetic field.

Learn more about The magnetic field here:- https://brainly.com/question/14411049

#SPJ4

An airliner arrives at the terminal, and the engines are shut off. The rotor of one of the engines has an initial clockwise angular speed of 1940 rad/s. The engine's rotation slows with an angular acceleration of magnitude 79.3 rad/s2. (a) Determine the angular speed after 10.0 s. (b) How long does it take the rotor to come to rest?

Answers

The angular speed after 10.0 seconds is approximately 2733 rad/s. it takes approximately 24.5 seconds for the rotor to come to rest.

In this problem, we are given the initial angular velocity of a rotor (1940 rad/s) and its angular acceleration (79.3 rad/s^2). We are asked to find the angular velocity after 10.0 seconds and the time it takes for the rotor to come to rest. To find the angular velocity after 10.0 seconds, we can use the formula omega_f = omega_i + alpha*t, where omega_f is the final angular velocity, omega_i is the initial angular velocity, alpha is the angular acceleration, and t is the time elapsed.

Plugging in the given values, we find that the final angular velocity is approximately 2733 rad/s. To find the time it takes for the rotor to come to rest, we set the final angular velocity to 0 and solve for t. We can use the same formula as before: omega_f = omega_i + alpha*t. Plugging in the values for omega_i and alpha, we find that it takes approximately 24.5 seconds for the rotor to come to rest.

In summary, the angular velocity of the rotor after 10.0 seconds is approximately 2733 rad/s, and it takes approximately 24.5 seconds for the rotor to come to rest.

To know more about Velocity

https://brainly.com/question/29852141

#SPJ4

A stone with a mass of 0.900 kg is attached to one end of a string 0.800 m long. The string will break if its tension exceeds 60.0 N. The stone is whirled in a horizontal circle on a frictionless tabletop; the other end of the string remains fixed.
Find the maximum speed the stone can attain without breaking the string.

Answers

The centripetal force is 7.3 m/s and tension  is 7.30 m/s .

What is tension ?

Any two physical objects in touch have the ability to exert forces on one another. Based on the kinds of objects in touch, we give these contact forces various names. When a rope, string, chain, or cable is one of the things applying the force, the force is referred to as tension.

What is speed ?

Speed is the rate of change in location of an item, expressed in metres per second. For instance, the speed of an item is one metre per second if it starts at the origin and goes three metres in three seconds. Speed is easily calculated by dividing a distance by a time.

Tension (force) in string = mv²/R = 0.90v²/0.80 = 1.125v²

Tension (force) in string = 60.0 N

set Tensions equal:

1.125v² = 53.33

v = v53.33

v = 7.30 m/s (speed of mass at which string breaks)

circumference of path mass travels = 2pR = 5.02 m

time of one rev that gives a tangential speed of 7.3 m

T = 5.02/7.3 = 0.69 sec

rev/sec = 1/0.69 = 1.45 rps

60 x 1.45 = 87 rpm

centripetal force = Tension

mv^2/r = T

0.9xv^2/0.8 =60

v=7.3 m/s

Therefore, the centripetal force is 7.3 m/s and tension  is 7.30 m/s .

Learn more about tension from the given link.

https://brainly.com/question/138724

#SPJ4

The figure below shows an overhead view of three particles on which external forces act. The magnitudes and directions of the forces on two of the particles are indicated.
The
figure below shows an overhead view of three p
(a) What are the magnitude and direction of the force acting on the third particle if the center of mass of the three-particle system is stationary?
8 N, rightward
8 N, leftward
2 N, rightward
2 N, leftward
(b) What are the magnitude and direction of the force acting on the third particle if the center of mass of the three-particle system is moving at a constant velocity rightward?
2 N, rightward
8 N, leftward
2 N, leftward
8 N, rightward
(c) What are the magnitude and direction of the force acting on the third particle if the center of mass of the three-particle system is accelerating rightward?
less than 2 N, rightward
greater than 2 N, rightward
greater than 2 N, leftward
less than 2 N, leftward

Answers

On two of the particles, the strengths and directions of the applied force are shown. 2 N to the right, 2 N to the right, and more than 2 N to the right

a) 2 N rightward

for the center of mass to be stationary,

Net force = 0

SO, 2N rightward

(b) 2N rightward

for the center of mass to move with constant velocity

Net force = 0

So, 2N rightward

(c) greater than 2N rightward

for the center of mass to accelerate in rightward

Net force should be in rightward

So, greater than 2N rightward

In mechanics, a force is any action that has the potential to change, maintain, or deform a body's motion. The three principles of motion outlined by Isaac Newton in his Principia Mathematica are frequently used to illustrate the concept of force (1687). Newton's first law states that a body at rest or moving uniformly in a straight line will stay in that state until a force is applied to it. According to the second law, a body will accelerate (change in velocity) in the direction of any external force acting on it. The strength of the external force directly correlates with the strength of the acceleration.

Learn more about force here:

https://brainly.com/question/13191643

#SPJ4

a plane electromagnetic wave is traveling in a vacuum. the magnetic field is directed along the z-axis and given by: B_z = 3.0 mu T sin(1.0 times 10^7 x - 3.0 times 10^15 t) True / False The wavelength lambda of the wave is 1.0 times 10^7 meters True / False The frequency f of the wave is 3.0 times 10^15 Hz. True / False The direction of the electric field is along the x-axis. True / False Doubling the magnetic field strength

Answers

In a vacuum, an electromagnetic wave is moving through the air. The wavelength is 1x10^7m and the frequency is 3x10^15Hz while the  direction of the electric field is along the z-axis.

Given the magnetic field along z axis (B) = 3.0muTsin(1.0x10^7x  - 3.0*10^15 t)

The formula for magnetic field  is B = B0sin(wt+kx)

For direction of wave by comparing the equations we have

kx +(- 3.0 x 10^15 ) = 0

x = 3.0 x 10^15 /k  in positive direction of x axis

The direction of propogation is along x axis and the magnetic field in y axis. The electric field is in both directions. So E along z axis

B0 = 3x10-7x4πT

ω = -3.0 x 10^15

∧ = 2π/k =  2x3.14/1.0x10^7 = 0.68/1.0x10^7 = 0.68x10-7

The wavelength ∧ of the wave is = 0.68x10-7m

frequency (f) = c/∧ = 3x10^8/0.68x10-7 = 4.41x10^15Hz

The frequency f of the wave is 4.41x10^15Hz

To learn more about magnetic field click here https://brainly.com/question/24397546

#SPJ4

a full moon appears larger on the horizon than it does overhead not because of the refraction of the atmosphere but because of an optical illusion. objects in the foreground influence the perceived size of objects in the background, and when the moon is on the horizon amid trees and houses the brain interprets it as being larger than when it appears overhead in the absence of those objects.

Answers

Moon Illusion is an Optical Phenomenon. This occurs because when the moon is near the horizon, Earthly things that resemble the moon on your retina generate a misunderstanding or inaccurate perception of distance and size.

The brain interprets the low moon as being bigger since it is lower in the sky than the higher moon.

The mechanism behind size-distance perception in everyday life, which transforms planar pictures that fall on the retina into a perception of hard objects moving in space, is related to the moon illusion.

What happens when the mechanism acts in an uncommon circumstance is what causes the moon illusion. In typical perception, the angular size of the light image stimulating our eyes expands or shrinks as rigid objects move in depth (distance).

The sense of hard objects with shifting depth perception is produced naturally by the brain in response to these changing stimuli. The ground and horizon make the moon appear comparatively close when it is close to the horizon.

Due to the moon's shifting apparent position in depth while the light stimulus stays constant, the moon appears extraordinarily huge due to the brain's size-distance mechanism.

To read more about optical phenomenons, visit https://brainly.com/question/28195646

#SPJ4

a current - carrying wire is pulled away from a conducting loop in the direction shown. As the wire is moving, is there a cw current around the loop, a ccw current or no current? Show the induced field and give clear explanation to your answer.

Answers

If the current increases continuously, then the direction of the induced current in the loop is clockwise.

A compass needle put above a straight wire carrying current points north (the magnetic south pole), hence the magnetic field at that location is towards geometric south, and using the right hand thumb rule, we can determine that the direction of the current is east. A counterclockwise current will be produced (ccw). The size of the loop shrinks as the bar ascends through the zone of constant magnetic field, and as a result, the flux through the loop also shrinks.

To learn more about magnetic field visit:https://brainly.com/question/14848188

#SPJ4

A stone is dropped from the top of a cliff. The splash it makes when striking the water below is heard 5.0 later.

Answers

The height of the cliff that a stone is dropped from the top, the splash it made as it hit the water below was heard 5.0 s later is 857.5 m.

Sound reflection

Sound can bounce when it hits an object that has a hard surface and is not easily penetrated by waves. Examples of objects that can reflect sound are glass walls, iron, zinc, cave walls, and seabeds.

This reflected sound can be used to measure the depth of the sea or the distance of the walls of the cave passage. One of the uses of sound reflection is to know the depth of the sea or the distance of the cave walls.

The equation to determine the distance is:

s = ½ (v) (t)

s = distance or height

t = time

v = velocity of the sound wave (343 m/s)

We have,

Time  = 5.0 s

Velocity of the sound wave = 343 m/s

So, the height of the cliff:

s = ½ (v) (t)

= ½ (343) (5)

= 857.5 m

The question is incomplete, it should be:

A stone is dropped from the top of a cliff. The splash it makes when striking the water below is heard 5.0 s later. How high is the cliff?

Learn more about sound wave here: https://brainly.com/question/1199084

#SPJ4

How does the inertia of the model car compare with the inertia of a real car

Answers

The mass of a real car is greater than that of a model car, hence it will have higher inertia.

What is inertia and give its example?

The concept of inertia states that an object will maintain its current motion unless a force changes its speed or direction. The phrase should be taken as a shortened form of Newton's first law of motion's description of "the principle of inertia."

Case Studies of the Law of Inertia in Daily Life (Inertia of Motion) People slump forward when the bus abruptly stops. The top part of the body continues to move forward due to inertia of motion when the driver of a bus abruptly brakes, but the bottom section of the body comes to rest along with the bus.

To learn more about inertia from given link

https://brainly.com/question/1140505

#SPJ1

white light falls on two narrow slits separated by 0.50 mm. the interference pattern is observed on a screen 5.5 m away.

Answers

The separation between the first maxima for red light (λ = 700nm) and violet light (λ = 400nm) = 80 x 10⁻⁵ cm

Light interference

Light interference is a condition when two or more light waves combine to form a combined light wave. The condition for this light interference is that the light waves come from a coherent source, so that the amplitude, wavelength, and frequency are the same, and the phase difference is always fixed.

The question is incomplete, it should be:

What is the separation between the first maxima for red light (λ = 700nm) and violet light (λ = 400nm)?

We have,

Distance between slits = 0,5 mm = 5 x 10⁻⁴ m

Distance between the slit and the screen = 5.5 m

λ red = 700 nm =

λ violet = 400 nm

So, The separation between the first maxima:

Sin ∅m = mλ/d

= (1 x 400 x 10⁻⁹)/5 x 10⁻⁴

= 80 x 10⁻⁵ cm

The question is incomplete, it should be:

White light falls on two narrow slits separated by 0.50 mm. The interference pattern is observed on a screen 5.5 m away. (a) What is the separation between the first maxima for red light (λ = 700 nm) and violet light (λ = 400 nm)?

Learn more about light interference here: https://brainly.com/question/17112134

#SPJ4

consider a diffuser operating in an adiabatic process. the device decelerates the flow, causing a decrease in the kinetic energy of the fluid. noting from the first law that energy cannot be destroyed, describe physically what happens to the lost kinetic energy.

Answers

A diffuser is an adiabatic device that slows down a fluid to reduce its kinetic energy.

The transfer of heat from one system to another is caused by heat, temperature, and external variables. When heat is transferred, things do change. A process is referred to as adiabatic if there is no heat transfer.

Consider the internal combustion engine (ICE) found in classic automobiles. The ICE won't be able to function without the air pressure generated in the combustion chamber to move the piston inside a cylinder. Similar to this, the steam engine, which was widely employed throughout the early stages of the Industrial Revolution, functions according to the same theory. Heat is delivered to a small space, which causes the wet air to expand and force cylinders to rise and fall.

To learn more about adiabatic please click on below link

https://brainly.com/question/14930930

#SPJ4

a parallel-plate capacitor is made from two plates 12.0 cm on each side and 4.50 mm apart. half of the space between these plates contains only air, but the other half is filled with plexiglas of dielectric constant 3.40 (figure 1). an 18.0 v battery is connected across the plates.

Answers

An 18.0 v battery is connected across the plates is 4.58 * 10^-9 J.

Ceq = A*eo/2*d * (K1 + K2)

Ceq = (0.12*0.12*8.85*10^-12)/(2*4.5*10^-3) * (1 + 3.4)

Ceq = 6.23 * 10^-11 F

Energy Stored = 1.2*cv^2

U = 1/2 * 6.23 * 10^-11 * 18.0^2 J

U = 1.0 * 10^-8 J

If we remove plexiglas,

Energy Stored = 1/2* cv^2

U = 1/2 * 2.832 * 10^-11 * 18.0^2 J

U = 4.58 * 10^-9 J

One or more electrochemical cells with external connections for powering electrical equipment make up an electric battery, which is a source of electric power. A battery's positive terminal functions as the cathode and its negative terminal as the anode while it is delivering electricity.

A battery is a device that uses an electrochemical oxidation-reduction (redox) cycle to turn the chemical energy included in its active components directly into electric energy. An electric circuit is used in this kind of reaction to transmit electrons from one substance to another.

To learn more about electric energy visit:https://brainly.com/question/16182853

#SPJ4

Three boxes are pushed on a floor with a constant force of magnitude F applied to box A. The boxes move to the right with an acceleration of 1.5 m/s2. The coefficient of kinetic friction between the bottom of a block and the floor is 0.4. 2.0 kg B 1.0 kg 3.0 kg What is the magnitude of the force that block B applies to block C? A) 1.5 N B) 4.5 N C) 7.3N D) 16.3 N E) There is not enough information to tell.

Answers

The magnitude of the force that block B applies to block C is 16.3  N .

Calculation :

Frictional force of this combination = 0.4*9.8(Ma + Mb)

                                                         = 0.4*9.8*3

                                                       F(ab)= 11.76 N

F(AB) = Net force of this combination = 3*1.5 = 4.5 N

since block ' c ' is also moving with the same acceleration ,

Net force on C by B = F - (F(ab) +F(AB) )

                                  = 32.52 - (11.76 + 4.5)

                                  = 32.52 - 16.26

                                   = 16.3 N

The word "force" has a precise meaning. At this level it is appropriate to describe the force as pushing or pulling. A force does not contain or "have within" an object. A force is applied from one object to another. The idea of force is not limited to animate and inanimate objects.

Learn more about force here : https://brainly.com/question/12785175

#SPJ4

The 2.7-cm-diameter solenoid in (Figure 1) passes through the center of a 6.0-cm-diameter loop. The magnetic field inside the solenoid is 0.30 T .
A. What is the magnetic flux through the loop when it is perpendicular to the solenoid?
B. What is the magnetic flux through the loop when it is tilted at a 60 angle?

Answers

A. The magnetic flux through the loop, when it is perpendicular to the solenoid, is approximately 8.49 Weber (Wb).

B. The magnetic flux through the loop, when it is tilted at a 60-degree angle, is approximately 4.24 Weber (Wb).

A:

To find the magnetic flux through the loop when it is perpendicular to the solenoid, we can use the formula for magnetic flux:
[tex]\phi=BAcos(\theta)[/tex]

The area of the loop is given by:
[tex]A=\pi(d/2)^2[/tex]

B = 0.30 T and d = 6.0 cm

The magnetic flux is given as:
[tex]\phi= 0.30T\times (\pi \times (6/2)^2\\\phi=8.49\ Wb[/tex]

B:
To find the magnetic flux through the loop when it is tilted at a 60-degree angle, we need to consider the new angle (θ = 60 degrees) between the magnetic field and the loop's normal vector.
The magnetic flux:

[tex]\phi=0.30*(\pi*(6/2)^2)cos(60^o)\\\phi=4.24\ Wb[/tex]

Learn more about magnetic flux here:

https://brainly.com/question/1596988

#SPJ12


which of the following is the statement of dalton's law of partial pressures? multiple choice question. pv

Answers

The statement of dalton's law of partial pressures is the total pressure by a mixture of gases is equal to the sum of the partial pressures of each gas that makes up the mixture. The correct answer is B.

The pressure that each gas would produce if it occupied the same volume of the mixture at the same temperature alone is referred to as the partial pressure.

the law of partial pressure known as Dalton's Law A combination of gases exerts a pressure that is equal to the sum of the pressures that each gas exerts on its own. Each gas exerts pressure directly proportional to the amount of that gas present in the gas mixture.

In closed vessels over water, Dalton found that oxygen interacted with either one or two volumes of nitric oxide. This ground-breaking observation of integral multiple proportions served as crucial experimental support for his fledgling atomic theories.

Your question is incomplete but most probably your full question was

Which of the following is a statement of daltons law of partial pressure

A. The total pressure of a mixture of gases is the product of the partial pressures of each gas in the mixture

B. The total pressure of a mixture of gases is the sum of the partial presaure of each gas that makes up the mixture

C. gases can exist as mixtures at STP only

D. in a mixture of two gases, the pressure of each gas is 1/2 of the total pressure

E. the partial pressure of a gas is equal to the total pressure of the mixture

Learn more about Dalton's law of partial pressures here:- brainly.com/question/14119417

#SPJ4

most of the new star formation in the galaxy is found in the most of the new star formation in the galaxy is found in the bulge. halo. galactic center. spiral arms. globular clusters.

Answers

"Most of the new star formation in the galaxy is found in the spiral arms."

Spiral arms are the regions of strong star formation and are more prominent in blue light. From the centre of spiral and barred spiral galaxies, these regions extend.

Transient weaknesses in the gases that encourage star formation and produce turbulence lead to spiral arms. In other words, spiral arms contain a lot of gases, which are the primary building blocks needed for the development of new stars.

They are not hard or permanent structures, as evidenced by the presence of spiral arms. A density wave is currently thought to circle a spiral galaxy, creating spiral arms.

The thin disk of the galaxy contains all of the spiral arms.

To know more about galaxies:

https://brainly.com/question/12532606

#SPJ4

if the breaking tension of the cord is 620 n, what is the maximum mass of the sign to put on without breaking the cord

Answers

The highest tensile stress that a material can sustain before failing, such as breaking or irreversible deformation, is known as the material's breaking strength. The point at which a material transitions from elastic to plastic deformation is defined by its tensile strength.

What is an example of breaking stress?

A metal has a breaking stress or strength of 1,000 pounds per square inch, for instance, if a metal rod with a one square inch cross section can take a pulling force of 1,000 pounds but breaks if additional force is applied.

How can I determine a string's breaking strength?

The breaking strength of the string in a basic pendulum is equal to twice the weight of the bob. When the string is horizontal, the bob is freed from its resting position. When the string forms an angle with the vertical, it snaps.

To know more about breaking strength visit;

https://brainly.com/question/14175453

#SPJ4

Other Questions
ella purchased a new car in 2000 for $27,600. the value of the car has been depreciating exponentially at a constant rate. if the value of the car was $8,300 in the year 2004, then what would be the predicted value of the car in the year 2009, to the nearest dollar? In which type of election doe the tate lit all of the available candidate from all the political partie on the ballot Resource typing is a _______ process designed to be as _______ as possible to facilitate frequent use and accuracy in obtaining needed resources.answer choices:continuous; simpledefined; rigidcomplex; broadlocal; regulated the nurse is planning care for a client with an open wound following surgery for a ruptured appendix. what short-term client goals help prepare the client for discharge? select all that apply. What animated film is about a train taking kids to the North Pole on Christmas Eve? at strut intersection headed eastbound, atc instructs you to hold west on the 10 dme fix west of lch on v306, standard turns. what entry procedure is recommended? a. teardrop. b. direct. c. parallel. Math Can you thech me about being a businessman and now its not even that bad I am going to mrs. pearson cut judy's hot dog into eight pieces and sylvia's into six pieces. sylvia cried because she felt she wasn't getting as much hot dog as judy. piaget would say that sylvia doesn't understand to principle ofconservation.object permanence.accommodation.egocentrism. Problem-solvinga How many codes can Eddie create usingEddie needs to choose a 6-digit code for his computer password.i 6 numbersii 4 numbers followed by 2 lettersiii 1 number followed by 5 letters?Eddie decides that he does not want to repeat a digit or a letter.b How many ways are possible in parts i to iii now? if the us government increases deficit spending, which of the following will occur as a result of the change in the interest rate? Alec is an American working for a software company in Texas. He has been assigned to work on a project in China for a year. He would be considered a(n) _____ in China.a. domicile residentb. expatriatec. host-country nationald. third-country national business provides historical, current, and predictive views of business operations and environments and gives organizations a competitive advantage in the marketplace. a. exploration b. indexing c. differentiation d. intelligence You have just accepted a position as human resources manager for a large firm. Your first task is to design the training program for the company. You decide to focus your training efforts on skills because these skills transferable to other firms. general are not specific are higher than its surroundings2) An object will normally be a net radiator of energy when its temperature isA) lower than its surroundings.B) higher than its surroundings.C) neither of these. Is this a function or not a function? How much energy (in kJ) is gained by iron when a 24.6 g sample is heated from 23.0C to 99.8C. The specific heat capacity of iron is 0.387 J/(g*C) In humans, brown eyes (B) are dominant over blue eyes (b). A brown-eyed woman who has a blue-eyed child would have the genotype:A. Bb.B. BBB.C. BbBb.D. bb.E. BB. What are some EXTERNAL factors for the fall of Rome ? Alejo is writing a memo to explain additional details about the new company policy on sick leave. Staffmembers already know about the change and are not happy about it.What should Alejo write first in his memo?OA) Reasons behind the change to help staff understand and accept it.B) The purpose of the memo, which is to convey specific information about the new policy.C) An apology for the change and a note that he understands why it is unpopular.D) A list of details about the new policy to clearly provide the purpose of the memo. i. describe (list) the events from the time calcium ions enter the terminal axon at the neuromuscular junction through muscle cell contraction.